Diagnosis+Psychopathology- Exam Qs

¡Supera tus tareas y exámenes ahora con Quizwiz!

Selye's General Adaptation Syndrome involves the processes termed: Select one: A.alarm, resistance, exhaustion. B.anticipation, defensiveness, exhaustion. C.alarm, stimulation, resistance. D.attention, accommodation, adaptation.

Correct Answer is: A This paradigm was an early attempt to explain how we react to stress. It postulates that after we become aware of the stressful situation (alarm), we summon our resources and meet the challenge (resist). After the trauma is over, we collapse (exhaustion).

According to Theodore Millon, people with Narcissistic Personality Disorder rely primarily on which of the following defense mechanisms? Select one: A.repression, rationalization, and projection B.repression, displacement, and sublimation C.rationalization, reaction formation, and sublimation D.identification, intellectualization, and displacement

Correct Answer is: A This is a difficult question unless you are a Theodore Millon fan. According to Millon, when narcissistic people experience personal failure and public humiliation, they resort to defense mechanisms -- first repression, and if that fails, then rationalization and projection.

A man with Korsakoff's syndrome would most likely have difficulty remembering: A.the name of someone he met 10 minutes ago B.how to spell the word "world" backwards C.the name of a high school friend D.how to dress himself properly

Correct Answer is: A Alcohol-Induced Persisting Amnestic Disorder, also known as Korsakoff's syndrome, is one of the most common causes of amnesia and is due to thiamine deficiency associated with prolonged heavy drinking. The memory impairment usually continues even if the alcohol use has stopped. Korsakoff's syndrome is manifested by an impaired ability to learn new information (anterograde amnesia) or an inability to recall previously learned information (retrograde amnesia). However, the greatest impairment is among more recent memories, while remote memories remain relatively intact.

Of the following, which would be the most important factor to keep in mind when deciding whether to prescribe tricyclic anti-depressants to a patient? A.the patient's suicide risk. B.the patient's history of treatment success or failure with tricyclics. C.whether or not the patient's symptoms are atypical. D.whether or not the patient's symptoms are primarily vegetative.

Correct Answer is: A All of these factors would be important to consider in deciding whether to prescribe tricyclics to a patient. However, safety considerations generally supercede all others; thus, choice "the patient's suicide risk" is the best answer. The risk of suicide in depressed patients must always be considered by physicians when writing prescriptions -- particularly for potentially dangerous drugs such as tricyclics.

Women diagnosed with Borderline Personality Disorder likely have similar symptoms to men diagnosed with _________________. Select one: A.Antisocial Personality Disorder B.Avoidant Personality Disorder C.Narcissistic Personality Disorder D.Histrionic Personality Disorder

Correct Answer is: A Borderline Personality Disorder has several symptoms in common with Antisocial Personality Disorder (i.e., impulsivity); however many of the behaviors are gender-related. Consequently, women are more likely to receive a diagnosis of Borderline Personality Disorder and men a diagnosis of Antisocial Personality Disorder.

Screening for cognitive deficits associated with Alzheimer's dementia and other conditions with the Mini Mental Status Exam (MMSE) tends to result in a disproportionate number of: Select one: A.false positives for African-American and other minorities. B.true positives for individuals from lower socioeconomic backgrounds. C.false negatives for African Americans and other minorities. D.true negatives for individuals with lower levels of education.

Correct Answer is: A Due to the tendency to overestimate cognitive deficits among African Americans and other minority groups, the MMSE results in a disproportionate number of false positives. Research indicates the MMSE also overestimates cognitive deficits for individuals with lower socioeconomic backgrounds and lower levels of education. When controlling for education level and socioeconomic status, research findings have been mixed on whether the racial bias persists or can be eliminated.

Generalized tonic-clonic seizures are associated with Select one: A.grand mal epilepsy. B.petit mal epilepsy. C.Jacksonian epilepsy. D.temporal lobe epilepsy.

Correct Answer is: A Generalized tonic-clonic seizure is the name for what was previously referred to as grand mal seizure; thus, this choice "grand mal epilepsy" is the best answer. These seizures involve episodes of violent shaking, during which the person becomes blue and stiff. There may also be an increase in heart rate and blood pressure. Generalized tonic-clonic seizures may last up to an hour, and afterward the person falls into a deep sleep.

A difference between tangentiality and circumstantiality is that: Select one: A.in circumstantiality, the person is delayed in reaching the point, whereas in tangentiality, the person avoids reaching the point at all. B.in tangentiality, the person is delayed in reaching the point, whereas in circumstantiality, the person avoids reaching the point at all. C.tangentiality is speech that is not understandable, due to the lack of a meaningful connection between words or sentences; by contrast, circumstantial speech is understandable. D.circumstantiality is speech that is not understandable, due to the lack of a meaningful connection between words or sentences; by contrast, tangential speech is understandable.

Correct Answer is: A If you were not familiar with the term "tangentiality," thinking about the phrase "going off on a tangent" might have helped you. Tangentiality is defined as responding to a question in an oblique or irrelevant way; in other words, the person avoids the point entirely. Circumstantiality, by contrast, is a pattern of speech that is delayed in reaching the point due to much irrelevant detail or many parenthetical remarks. Eventually, however, the point is reached.

A person with mild Alzheimer's dementia would most likely obtain the lowest score on which of the following WAIS-IV Indexes? Select one: A.Processing Speed B.Perceptual Reasoning C.Working Memory D.Verbal Comprehension

Correct Answer is: A Individuals with probable or mild Alzheimer's dementia typically obtain the lowest scores on the Symbol Search and Coding subtests which are the core subtests for the Processing Speed Index.

Untreated diabetes mellitus can result in which of the following symptoms? Select one: A.increased appetite with weight loss, confusion, mental dullness, and apathy B.emotional lability, memory loss, depression, obesity C.accelerated heart rate, agitation, nervousness, fatigue, insomnia D.slowed heart rate, depression, lethargy, impaired concentration and memo

Correct Answer is: A Insulin, which is released by the pancreas, is involved in the uptake and use of glucose and amino acids. Hypoinsulinism produces diabetes mellitus (excessive blood glucose). When untreated, diabetes mellitus results in increased appetite with weight loss, apathy, confusion, mental dullness, polyuria, polydipsia, and increased susceptibility to infection. emotional lability, memory loss, depression, obesity The symptoms listed in this option are associated with Cushing's disease, which results from hypersecretion of cortisol. accelerated heart rate, agitation, nervousness, fatigue, insomnia The symptoms listed in this option are associated with hyperthyroidism, or Grave's disease (other symptoms of this disorder include a speeded up metabolism, elevated body temperature, heat intolerance, and increased appetite with weight loss). slowed heart rate, depression, lethargy, impaired concentration and memory The symptoms listed in this option are associated with hypothyroidism (other symptoms of this disorder include a slowed metabolism, reduced appetite with weight gain, lowered body temperature, and decreased libido).

The prevalence rate for autism spectrum disorder in epidemiological studies since 2000 indicate a convergence in the range of: Select one: A.60/10,000 B.27.5/10,000 C.10/10,000 D.4/10,000

Correct Answer is: A It appears the prevalence rate for autism spectrum disorder has increased worldwide over the past decade however whether there is an increased incidence of autism has yet to be determined partially due to confounding changes in diagnostic classification, assessment, and varied methodologies of surveys. In epidemiologic studies done in the 1960s, autism rates were typically based on "classical autism" or severe impairment of language, social interaction, and behavior. The studies estimated a prevalence rate of 4/10,000. Prevalence rates have been influenced by changes in diagnostic criteria, increased awareness of developmental problems, and improved assessment. A review of 32 epidemiologic studies done in 13 countries from 1987 to 2000 indicates the best estimate for the prevalence rate of autism spectrum disorder being 10/10,000. Epidemiological studies since 2000 indicate a convergence in the range of 60/10,000.

Dementia and depression both involve cognitive deficits. However, in comparison to dementia, depression: A.is more likely to involve impaired free recall while recognition memory is left relatively intact. B.is more likely to involve impaired declarative memory while procedural memory is left relatively intact. C.is more likely to involve impaired nonverbal memory than verbal memory. D.is more likely to cause deficits on abstract memory tasks than on concrete memory tasks.

Correct Answer is: A It's sometimes hard to distinguish between depression and dementia because they share several symptoms, especially cognitive deficits. However, the pattern of cognitive deficits differs. For example, depression is likely to impair free recall only, while dementia affects both free recall and recognition.

Which of the following is not an iatrogenic stressor for elderly clients? Select one: A.loss of support and companionship B.trouble reading directions on medicine labels C.illness due to medication being over prescribed D.drug interactions from multiple prescriptions from more than one physician

Correct Answer is: A Loss of support and companionship is a stressor, but not an iatrogenic stressor. An iatrogenic condition is one that is produced by the treatment. Some conditions are obviously iatrogenic in nature, such as a practitioner who inadvertently uses a medical remedy to which the patient is allergic or administers an erroneous medication dose. Others forms of iatrogenesis are less obvious, like symptoms arising in response to therapists' suggestions, e.g., recovered memory syndrome, Dissociative Identity Disorder (DID) or unproven medical/psychological treatments.

Which of the following best describes the results of studies investigating the role of psychosocial stressors in the course of Major Depressive Disorder, Recurrent? Select one: A.psychosocial stressors typically play a more significant role in triggering the first or second episodes than subsequent episodes. B.psychosocial stressors typically play a more significant role in triggering episodes late in the course of the disorder than the first two episodes. C.psychosocial stressors play a similar role in triggering episodes early and late in the course of the disorder. D.psychosocial stressors do not play a role in the disorder since, by definition, Major Depressive Disorder is not triggered by external events.

Correct Answer is: A Major Depressive Disorder, Recurrent involves two or more Major Depressive Episodes, which are characterized by five or more specific symptoms of depression present during the same two week period and representing a change from the previous level of functioning. Individuals with this disorder very commonly have more than two Major Depressive Episodes in the course of their lifetimes, and studies suggest that psychosocial stressors are more likely to trigger an episode early in the course of the disorder than they are later on.

Masters and Johnson found that their version of sex therapy, which incorporates education about sexuality, training in communication skills, and the technique known as sensate focus, is most effective for treating: Select one: A.premature ejaculation. B.impotence. C.sexual aversion. D.orgasmic disorder.

Correct Answer is: A Masters and Johnson found that close to 100% of individuals with premature ejaculation were helped by their program which incorporated education about sexuality, improving communication, and sensate focus.

Depression is often accompanied by memory deficits and other cognitive problems. Memory deficits most often involve: Select one: A.recall memory. B.recognition memory. C.implicit memory. D.recall and recognition memory but not implicit memory.

Correct Answer is: A Memory impairment is one of the characteristics that is useful for distinguishing depression (pseudodementia) from dementia. In depression, recall memory is affected but recognition memory is not; in dementia, recall and recognition memory are both impaired.

Research on epilepsy has found that tonic-clonic seizures are associated with: Select one: A.greater cognitive impairment than other types of seizures. B.less cognitive impairment than other types of seizures. C.cognitive impairment only when seizures are idiopathic. D.cognitive impairment only when seizures are secondary to known pathology.

Correct Answer is: A One of the complications of epilepsy is that repeated seizures are correlated, in some cases, with cognitive declines. This is especially true of tonic-clonic seizures regardless of whether they are idiopathic or secondary to (symptomatic of) another disorder.

Which of the following disorders is most likely to respond to pharmacotherapy alone? Select one: A.Bipolar I Disorder. B.Major Depressive Disorder C.Obsessive-Compulsive Disorder. D.Anorexia Nervosa.

Correct Answer is: A Pharmacotherapy (specifically, Lithium therapy in most cases) is the treatment of choice for manic-depressive illnesses such as Bipolar I Disorder. Pharmacotherapy may be supplemented with adjunctive psychotherapy (e.g., to provide support and coping skills). However, psychotherapy is not considered that useful in treating the core symptoms of Bipolar I Disorder. By contrast, although the disorders in the other choices are commonly treated with medication, they also respond to psychotherapy.

Which of the following tests would be most appropriate when assessing intelligence of a 10-year-old non-English speaking child? A.Raven's Color Progressive Matrices B.Roberts' Apperception Test for Children C.The Stroop Color Test D.Vineland Adaptive Behavior Scale

Correct Answer is: A Raven's Colored Progressive Matrices is a non-verbal intelligence test for ages 5 through 11 and is considered appropriate for non-English speaking children. Roberts' Apperception Test for Children (RATC) is a projective test which requires verbal responses and was not designed as a measure of intelligence. The Stroop Color-Word Test is a screening devise for neurological damage and relies on verbal identification of the words "red", "blue", and "green" which are printed in different colored inks. Finally, the Vineland is used to assess one's ability to care for oneself.

Research on the genetic contribution to major depression indicates the risk for depression for biological offspring is: Select one: A.essentially the same whether they have one or two parents with depression. B.higher if they have one parent with depression than if they have two parents with depression. C.higher if they have two parents with depression than if they have only one parent with depression. D.higher if they have a mother with depression than if they have a father with depression.

Correct Answer is: A Research findings show having a biological parent with depression increases an offspring's risk for major depression. What may be surprising however is that the risk is similar whether one or both parents experience major depression. Studies also indicate maternal and paternal depression affect biological offspring similarly in terms of rates of major depression.

The most consistent predictors of adolescent suicide are: Select one: A.Depression, use of drugs and alcohol, antisocial behavior B.Depression, loss of popularity, avoidance of social situations C.Use of drugs and alcohol, body weight, level of social acceptance D.Use of drugs, cigarette smoking, depression

Correct Answer is: A Research on risk factors is not entirely consistent. However, most studies concur with Gould (1992) who found the most consistent predictors to be a diagnosis of depression, use of drugs and alcohol, and antisocial behavior.

Chronic alcoholism may cause cognitive impairments. Specifically, following prolonged, heavy drinking, a person is most likely to exhibit: Select one: A.greater deficits in visuospatial skills than in verbal skills. B.greater deficits in verbal skills than in visuospatial skills. C.a comparable pattern of deficits in visuospatial and verbal skills. D.deficits in either visuospatial or verbal skills, depending on whether the person is left- or right-brain dominant.

Correct Answer is: A Studies looking at cognitive impairment due to prolonged, heavy alcohol use have found that it is more likely to involve problems in visuospatial skills than in verbal skills.

A measurement scale often used in rehabilitation settings to indicate a patient's level of response and ability to function is the Rancho Los Amigos scale, named for the rehabilitation hospital where it was created in California. Using this scale, a person recovering from a brain injury is most likely to have a low score: A.when admitted. B.at time of discharge. C.12 months after discharge. D.several years after discharge.

Correct Answer is: A The Rancho Los Amigos Scale provides a descriptive guideline of the various stages a brain injury patient will experience as he/she progresses through recovery and is most helpful in assessing the patient in the first weeks or months following an injury because it is based on observations of the patient's response to external stimuli and does not require cooperation from the patient. The Rancho scale is often administered in acute rehabilitative settings following release from intensive care and evaluates eight levels of functioning.

The findings of the National Institute of Mental Health (NIMH) research project regarding the relative effectiveness of cognitive-behavioral therapy and antidepressant medication in the treatment of depression indicate that Select one: A.both treatments are about equally effective. B.cognitive/behavioral therapy is more effective in the short-term, while medication is more effective in the long-term. C.cognitive-behavioral therapy has an immediate effect, while medication has a delayed effect. D.cognitive-behavioral therapy is more effective.

Correct Answer is: A The findings of the NIMH research project were that, on an overall level, cognitive-behavioral therapy (CBT) and antidepressant medication are about equally effective in the treatment of depression. There was some evidence that the treatments differed in the types of depression for which they are most effective. Specifically, some evidence suggested that CBT is more effective for milder forms of depression, while medication is more effective for relatively severe forms. But in terms of overall effectiveness, both treatments were found to be about equal.

For women in general, the typical pattern of depression and dysfunctional physical symptoms accompanying a menstrual cycle is that: Select one: A.there is no typical pattern. B.the symptoms occur shortly before and terminate shortly after the onset of menses. C.the symptoms occur shortly after the onset of menses. D.the symptoms occur and terminate shortly before the onset of menses.

Correct Answer is: A The key words in this question were "for women in general." You had to know that PMS is actually not typical of most women. While it is not uncommon for women to have mild psychological symptoms during the latter part of the luteal phase, there is no specific pattern typical of most women. Make sure you read each word in the question. the symptoms occur shortly before and terminate shortly after the onset of menses. If the question were about women who do suffer from PMS, this would be the correct answer.

The Personality Disorder that has been found to have the best prognosis is: Select one: A.Borderline Personality Disorder B.Paranoid Personality Disorder C.Dependent Personality Disorder D.Obsessive-Compulsive Personality Disorder

Correct Answer is: A The majority of individuals diagnosed with Borderline Personality Disorder (BPD) demonstrate significant reduction or remission of symptoms by middle age or sooner. Longitudinal studies involving those diagnosed in adolescence or early adulthood indicate that symptom resolution with impulsive symptoms are the quickest to resolve, followed by cognitive and interpersonal symptoms and finally affective symptoms are the most chronic and show the least improvement with increasing age.

Research on gender differences in anxiety has most often found that: Select one: A.females have more general anxiety than males B.males have more general anxiety than females C.females have more social and general anxiety than males D.males have more social and general anxiety than females

Correct Answer is: A The results of a meta-analysis on gender differences concluded that females (across all age groups) have a higher level of general anxiety than males. Males scored very slightly higher than females in level of social anxiety, although this difference was not significant

General symptoms that may accompany the third stage of Alzheimer's Dementia include Select one: A.apathy and emotional blunting. B.depression and anomia. C.irritability and anger. D.paranoia and labile mood.

Correct Answer is: A These symptoms are characteristic of the third stage of Alzheimer's Dementia according to many authors. The symptoms vary considerably from person to person and may include personality, behavioral, and cognitive changes. The symptoms of depression and anomia* and irritability and anger* are often seen during the first stage. Paranoia and labile mood* are characteristic of the second.

Of the following, which would be the most effective treatment for Factitious Disorder? Select one: A.outpatient treatment using supportive therapy B.outpatient group or family therapy C.inpatient individual and group therapy D.inpatient treatment using confrontational therapy

Correct Answer is: A Treatment for Factitious Disorder typically involves symptom management rather than eliminating or "curing" the disorder, which is associated with poor therapeutic outcome. In absence of a consistently effective treatment, establishment of a strong therapeutic relationship, use of supportive therapy and consistency of care is considered the best approach for managing Factitious Disorder symptoms. Family and group therapy can be helpful for family members coping with the client's symptoms, but it is not the most effective treatment for Factitious Disorder. Inpatient treatment is generally contraindicated given the client's underlying need "to adopt the sick role" and use of confrontational techniques, with the risk of client defensiveness, denial or therapeutic relationship termination, requires caution in the treatment of Factitious Disorder

Based on the Centers for Disease Control (2005) statistics, the suicide rate is highest for males between the ages of: Select one: A.75-79. B.40-44. C.20-24. D.15-19.

Correct Answer is: A While suicide rates tend to vary from year to year, a good overall generalization is that the highest suicide rates are for older men. In this report, the rate was 31.85 for 2005 and 33.11 for 2000 through 2005 for males ages 75-79. For males ages 40-44, the reported rate was 24.67 for 2005 and 24.25 for the years 2000 through 2005; For ages 20-24, the reported rate was 20.22 for 2005 and 20.62 for the years 2000 through 2005; and for males ages 15-19, the rate was 12.06 for 2005 and a rate of 12.39 for the years 2000 through 2005.

A friend of yours was involved in a car accident three weeks ago in which the other driver was killed. She tells you she needs to see you for therapy and that she is having dreams about the accident. She has been unable to drive or ride in a car and she tells you that "she feels nothing" about the accident. You refer her to another competent therapist who will probably diagnose your friend with Select one: A.Acute Stress Disorder. B.Post-traumatic Stress Disorder. C.Specific Phobia. D.Generalized Anxiety Disorder.

Correct Answer is: A Your friend will be diagnosed with Acute Stress Disorder. She has witnessed a traumatic event and is reexperiencing it through dreams. She is avoiding activities associated with the trauma and is experiencing a general numbing of feeling. For "Post-traumatic Stress Disorder," the same symptoms would need to continue beyond one month. For "Specific Phobia," while your friend may have phobic symptoms, it is related to a specific traumatic event rather than all cars, and for "Generalized Anxiety Disorder," again the symptoms have only been occurring for a few weeks and are specific to the traumatic situation.

Which of the following factors is associated with the worst prognosis for Schizophrenia? Select one: A.male, positive symptoms, and late onset B.male, negative symptoms, and early onset C.female, negative symptoms, and early onset D.female, positive symptoms, and late onset

Correct Answer is: B A number of factors have been associated with a better prognosis in Schizophrenia. These include being female, a later age of onset, and positive symptoms (which are more responsive to medications), as well as good premorbid adjustment, acute onset, precipitating events, treatment with antipsychotic medication soon after onset of illness, consistent medication compliance, and no family history of Schizophrenia.

The suicide rate for African-American adolescent males over the past few decades has been: Select one: A.higher than the rate for white adolescent males, but there has recently been an increase in the rates for both groups. B.lower than the rate for white adolescents males, but there has been an increase in the rates for both groups. C.higher than the rate for white adolescent males, but there has recently been a decrease in the rates for both groups. D.lower than the rate for white adolescent males, but there has been a decrease in the rates for both groups.

Correct Answer is: B Across all age groups and both genders, the suicide rate is higher for whites than for African-Americans.

Which of the following groups has the highest rate of completed suicide? Select one: A.Caucasian females B.Caucasian males C.African-American females D.African-American males

Correct Answer is: B Across all age groups, Caucasian males have a higher rate of completed suicide that the other groups listed by the choices.

While Sleep Terror Disorder and Sleepwalking Disorder are similar in terms of sleep and EEG patterns, there are behavioral differences that distinguish them. Sleepwalking Disorder, in contrast to Sleep Terror Disorder,: Select one: A.is accompanied by high levels of autonomic arousal during the episode B.is associated with prominent, organized motor activity during the episode C.is not associated with amnesia for the episode when the individual awakens in the morning D.is not associated with a family history of Sleepwalking and/or Sleep Terror Disorder

Correct Answer is: B Both Sleep Terror Disorder and Sleepwalking Disorder have been linked to a family history and are associated with amnesia for the episode upon awaking in the morning. Sleepwalking Disorder is usually associated with low levels of autonomic arousal and is characterized by prominent, organized motor activity such as walking around, talking, and eating. Both disorders do involve motor activity, although the activity associated with Sleep Terror Disorder is less organized and usually involves resisting being touched or held and sitting up.

A psychologist refers a client with bulimia nervosa to a physician for a medical evaluation. The physician discovers that the client has a serious medical complication associated with binging and purging, involving a low level of serum potassium. The client is at risk for kidney failure and cardiac arrest. This condition is called: Select one: A.hypoglycemia B.hypokalemia C.hyperorexia D.hypalgesia

Correct Answer is: B Certain behaviors associated with bulimia, specifically frequent vomiting and laxative use, can lead to serious medical complications, including electrolyte disturbances. Potassium is an electrolyte that is important to the function of the nerve and muscle cells, including the heart. Hypokalemia is the name given to the condition that involves low levels of serum potassium. Hypoglycemia* is characterized by less than normal amounts of glucose in the blood. Hyperorexia* refers to excessive appetite and hypalgesia* involves a relative insensitivity to pain (* incorrect options).

When applied to substance dependence, the specifier "early full remission" means that the client has been without any symptoms of substance abuse or dependence: Select one: A.for at least six months but less than one year B.for at least three months but less than one year C.for at least two weeks but less than one year D.for at least one year

Correct Answer is: B Early full remission, early partial remission, sustained full remission, or sustained partial remission are specifiers used to describe a substance dependence diagnosis. "Early" means that there is more than one month but less than 12 months of remission. "Sustained" means that there is 12 months or longer of remission. "Full" means that the person no longer meets any of the criteria for substance dependence or abuse, and "partial" means that one or more of the criteria for substance dependence are still met but the full criteria are no longer met.

The most serious potential medical side effect of Bulimia Nervosa is Select one: A.dehydration. B.electrolyte imbalance. C.fatigue. D.malnutrition.

Correct Answer is: B Electrolytes are salts that conduct electricity; they are found in the body fluid, tissue, and blood. Examples are chloride, calcium, magnesium, sodium, and potassium. Electrolyte imbalance can be caused by the frequent induced vomiting and overuse of laxatives that occur in Bulimia Nervosa. The most frequently found electrolyte disturbance in Bulimia is hypokalemia, a reduced level of potassium in the blood. Electrolyte imbalances can cause cardiac irregularities and cardiac arrest, potentially leading to death. Dehydration and fatigue are also potential side effects of Bulimia, but their potential consequences are not as serious as those of electrolyte imbalance.

From Wolpe's classical conditioning perspective, neurotic depression: Select one: A.is a conditioned response that can be alleviated through extinction trials in which the neutral (conditioned) stimulus is repeatedly presented without the depression-inducing (unconditioned) stimulus. B.is a response to anxiety and can, therefore, be alleviated by using systematic desensitization to eliminate the anxiety. C.is due to attributional biases that, through conditioning, have become associated with certain types of events and can be eliminated through reattribution training. D.results when there is an absence of response contingent reinforcement and is best treated by counterconditioning in which depression is paired with a variety of pleasure-producing (unconditioned) stimuli.

Correct Answer is: B Even if you are unfamiliar with Wolpe's explanation of depression, you may have been able to pick the right answer to this question as long as you have him associated with systematic desensitization. Wolpe distinguished between several types of depression. He linked neurotic depression to anxiety and considered systematic desensitization to be an effective treatment.

Research suggests that the best single treatment for Agoraphobia is: Select one: A.implosive therapy. B.flooding. C.systematic desensitization. D.psychotropic medication.

Correct Answer is: B Flooding involves exposing an individual to anxiety-provoking stimuli while preventing an avoidance response. In-vivo flooding is considered the most effective psychological treatment for Agoraphobia, with reports of long-term improvement for to 75% of treated patients.

During a grand mal seizure, the clonic phase involves Select one: A.minor abrupt movements of the eyelids, facial muscles, and arms. B.violent rhythmic contractions of the extremities. C.extension of the extremities. D.nonrhythmic jerks of the head, limbs, and trunk.

Correct Answer is: B Grand mal seizures involve a tonic phase (extension of the limbs), which is then followed by a clonic phase (violent rhythmic contractions).

Hypnosis would least likely be used in the treatment of: Select one: A.Substance Abuse. B.Obsessive-Compulsive Disorder. C.Specific Phobia. D.chronic pain.

Correct Answer is: B Hypnosis is contraindicated in individuals who have difficulty giving up control, such as obsessive-compulsive patients, or those who have difficulty with basic trust, such as paranoid patients. By contrast, hypnosis is often employed in the treatment of the other disorders. In the treatment of Substance Abuse, varying degrees of success have been reported; however, it continues to be used. It is also commonly used to help induce relaxation and/or gain access to painful memories in treating individuals with Anxiety Disorders; moreover, according to some, it is particularly effective in treating individuals with Specific Phobia because they are highly hypnotizable. Finally, hypnosis is also commonly used in the treatment of medical conditions that have a psychological component such as asthma, chronic pain, obesity, etc.

An examinee with traumatic brain injury (TBI) is most likely to obtain the lowest score on which of the following WAIS-IV Indexes? Select one: A.Perceptual Reasoning B.Processing Speed C.Verbal Comprehension D.Working Memory

Correct Answer is: B Individuals with TBI are most likely to obtain the lowest score on the Processing Speed Index and the highest score on the Verbal Comprehension Index.

Korsakoff's Syndrome is characterized by: Select one: A.anterograde amnesia without retrograde amnesia. B.anterograde amnesia and retrograde amnesia that involves a temporal gradient in which remote events are recalled better than recent events. C.anterograde amnesia and retrograde amnesia that involves a temporal gradient in which recent events are recalled better than remote events. D.anterograde amnesia and "flat" retrograde amnesia in which deficits in remote and recent memory are about the same.

Correct Answer is: B Korsakoff's Syndrome involves both anterograde and retrograde amnesia. The retrograde amnesia usually affects recent long-term memories more than remote memories (i.e., the person can recall events from early childhood but not from the last decade or two).

A psychologist asks a series of questions to determine whether the client has been experiencing vegetative symptoms when assessing a client who reports feeling depressed. Which of the following are vegetative symptoms? A.psychomotor retardation, appetite changes, social withdrawal B.sleep difficulties, appetite changes, psychomotor retardation C.social withdrawal, loss of concentration, reduced energy level D.confusion, psychomotor agitation, sleep difficulties

Correct Answer is: B Many clients may report conditions that suggest vegetative symptoms associated with a mental disorder. The classic, or "vegetative," signs of depression include persistent problems with appetite, weight loss or gain, sleep difficulties, reduced energy level, and changes in sexual desire or function. These symptoms are mostly objective, as opposed to subjective and can serve as useful data for diagnostic screening purposes as vegetative symptoms sometimes suggest a serious mental disorder. In particular, it is useful to identify whether any of these symptoms reflects a change from the client's previous functioning.

According to Marlatt's theory of substance dependence: Select one: A.a relapse of dependence is likely if the person makes non-dispositional attributions for use following a "slip" (use of a drug after a period of abstinence). B.substance use is "over-learned" in that it is maintained by itself as well as by multiple cognitive mediators and external reinforcers. C.addictions can be easily extinguished. D.excessive substance use is related to an unresolved need for power.

Correct Answer is: B Marlatt's model of substance dependence holds that addicts learn to associate substance use with relief of self-criticism and guilt through a variety of cues and reinforcers, such as advertisements depicting people feeling cheerful when drinking and social occasions in which a carefree attitude is reinforced. In other words, there are a variety of cues and mediators that serve to encourage and reinforce use. Contrary to the person's expectations, however, excessive substance use only exacerbates problems, such as interpersonal or work-related problems. This leads to more self-criticism and guilt, which the person again attempts to relieve by using. In other words, substance use is self-reinforcing -- it is the cause of and the expected solution to the same problems. And it is "over-learned" in that, due to the multiplicity of its antecedents, it becomes a strongly ingrained behavior. a relapse of dependence is likely if the person makes non-dispositional attributions for use following a "slip" (use of a drug after a period of abstinence). You might have gone for this option, since you probably knew that Marlatt is associated with a theory of relapse prevention that has to do with attributions regarding the reasons for "slips." However, according to Marlatt, a slip is likely to lead to a full-blown relapse when the person makes dispositional attributions for it, such as when the person blames him or herself. Relapse prevention involves teaching the person to make non-dispositional attributions, such as blaming the situation or the nature of the disease. So this question illustrates the importance of reading carefully and processing what you read, rather than relying on the recognition of "buzzwords."

Depressive episodes that occur in a seasonal pattern most likely include: Select one: A.loss of weight B.increased sleep C.increased energy D.increased craving for protein-rich foods

Correct Answer is: B Mood Disorder With Seasonal Pattern (better known as Seasonal Affective Disorder) is diagnosed when the person displays onset and remission of Major Depressive Episodes at characteristic times of the year. In most cases, onset is in fall or winter and remission is in spring. The depressive symptoms in Seasonal Affective Disorder most often include: decreased energy, increased sleep, overeating, weight gain, and a craving for carbohydrates.

The incidence of Obsessive-Compulsive Disorder begins to differ for males and females at which age: Select one: A.3 years B.6 years C.12 years D.18 years

Correct Answer is: B Obsessive-Compulsive Disorder has an earlier peak onset for males than females. For males the peak onset is between ages 6 and 15, and for females it is between ages 20 and 29. Thus, gender differences for OCD begin to become apparent at 6 years. However, in adulthood the incidence is about the same for both genders.

A 50-year old has memory loss as the result of her long-term heavy alcohol consumption. Most likely, she has trouble Select one: A.getting dressed. B.remembering someone she met a week ago. C.recalling her first date. D.repeating five digits forward and backward.

Correct Answer is: B People with Korsakoff's Syndrome (amnesia due to heavy, long-term alcohol consumption) have severe anterograde amnesia along with retrograde amnesia for events that occurred in the relatively recent past. getting dressed. Procedural memory is undisturbed. recalling her first date. Remote memories are usually intact. repeating five digits forward and backward. Short-term memory is unaffected.

When a person with deep dyslexia is presented with the written word "cat," he is most likely to respond with which of the following? Select one: A.tac B.dog C.at D.I don't know

Correct Answer is: B People with deep dyslexia exhibit a number of reading errors including semantic paralexia, which involves producing a response that is similar in meaning to the target word (e.g., dog for cat or arm for leg).

A family with an elderly parent who is in the early stages of Alzheimer's Dementia presents for a initial consultation. The best intervention on the part of a psychologist would be to Select one: A.work with the elderly patient on memory exercises. B.refer the family to a support group. C.begin family therapy to explore how the patient's diagnosis has affected family subsystems. D.work with the identified patient in individual therapy.

Correct Answer is: B Psychological treatment of Alzheimer's focuses primarily on optimizing the patient's immediate environment and providing support for the patient and his or her family. Both these goals can be met at least to some extent through a referral to a support group.

Reactive Attachment Disorder involves Select one: A.inappropriate interactions with unfamiliar adults. B.emotionally withdrawn behavior toward adult caregivers. C.over-attachment to a primary caretaker and fear of others. D.symptoms of Posttraumatic Stress Disorder in children.

Correct Answer is: B Reactive Attachment Disorder occurs in children below the age of 5 and is characterized by disturbed social relatedness; emotionally withdrawn behavior toward adult caregivers as manifested by a lack of seeking or responding to comfort when distressed.

Among adults who were diagnosed with ADHD during childhood, we could expect that Select one: A.their children will have a 25% chance of developing ADHD B.up to 70% will continue to exhibit signs of the disorder throughout their lives C.the symptoms will manifest in the same ways in those who continue to have the disorder D.by the time they reach adulthood, they will not exhibit further symptoms of the disorder

Correct Answer is: B Research studies have found that between 30% and 70% of children with ADHD continue to exhibit signs of the disorder throughout their lives. Studies have also found a strong genetic basis for ADHD. One study found that the risk of a child developing ADHD is 57% if one of their parents has the disorder

Patients with Schizophrenia are most likely to have: Select one: A.an enlarged hippocampus. B.enlarged ventricles. C.enlarged frontal lobes. D.extreme hemispheric asymmetry.

Correct Answer is: B Schizophrenia has been linked to several structural brain abnormalities, but the most consistent finding is that it is related to enlarged lateral and third ventricles. Lifetime Risk (%) Unrelated 1% Biological Siblings 10% Dizygotic (Fraternal) Twins 16% Monozygotic (Identical) Twins 48% Mednick (1958) proposed a theory which posits that Schizophrenia is a disorder occurring in individuals physiologically predisposed to this disorder who are confronted with an adverse and stressful environment. This is termed the diathesis-stress, or vulnerability, theory.

The incidence of OCD is: A.the same for males and females across all ages B.about twice as common in males than females C.higher among boys but about equal among adult males and females D.higher among girls but about equal among adult males and females

Correct Answer is: C Obsessive-Compulsive Disorder has an earlier peak onset for males than females. For males the peak onset is between ages 6 and 15, and for females it is between ages 20 and 29. The incidence is, therefore, higher among boys than girls but in adulthood the incidence is about the same.

During a board meeting, Dave's co-worker presented and took sole credit for ideas and work that Dave had done. Dave initially felt furious but forced himself to calm down and control his reaction, despite still being very upset, so as not to appear unprofessional in front of the board members. According to Selye's GAS, Dave was in the __________ phase. A.recovery B.resistance C.exhaustion D.alarm

Correct Answer is: B Selye's general adaptation syndrome (GAS) attempts to explain stress reactions and characteristic responses under conditions of stress. It postulates that after a person becomes aware of the stressful situation (alarm), they summon their resources and meet the challenge (resistance). After the trauma is over, they collapse (exhaustion).

A person with Somatization Disorder is most likely to also have which of the following diagnoses: Select one: A.a Substance Abuse Disorder B.a Personality Disorder C.Schizophrenia D.Panic Disorder

Correct Answer is: B Somatization Disorder has several frequently occurring comorbid conditions. Research indicates that 61% of Somatization Disorder patients have one or more co-occurring personality disorders. The next most frequent co-diagnoses are Major Depression (55%), Generalized Anxiety Disorder (34%), and panic disorder (26%). Substance-Related Disorders are also frequently associated with Somatization Disorder, however, not as frequently as the above diagnoses.

Recent research on gender affirmation surgery outcomes, following surgery, indicates gender dysphoria: A.continues to be experienced by a majority of individuals. B.is no longer experienced by a majority of individuals. C.is increasingly experienced by a majority of individuals. D.is no longer experienced by a majority of male-to-female patients, whereas female-to-male patients experienced an increase in gender dysphoria.

Correct Answer is: B Studies of individuals who have undergone gender affirmation surgery have found the vast majority no longer expressed gender dysphoria and were psychologically, socially and sexually functioning well following surgery. Smith et al. (2005) found, post-operatively, female-to-male and homosexual transgender people functioned better in many respects than male-to-female and straight transgender and straight individuals with greater psychopathology and body dissatisfaction reported the worst post-operative outcomes.

Briquet's syndrome is also known as a: Select one: A.Conversion Disorder B.Somatization Disorder C.Body Dysmorphic Disorder D.Hypochondriasis

Correct Answer is: B Termed after the physican who described the condition in the 1850s, Briquet's syndrome, or Somatization Disorder, is a chronic Somatoform Disorder with multiple physical symptoms that cannot be explained entirely by a general medical condition or the effects of a substance. The other three response choices are also Somatoform Disorders.

Possible causes of a Mood Disorder caused by a known organic factor such as a medical condition or substance use include: Select one: A.PCP use, cataracts, and ulcer B.viral illness, hallucinogen use, and carcinoma of the pancreas C.barbiturate use, hypothyroidism, and broken bones D.hyperthyroidism, cerebral palsy, and arthritis

Correct Answer is: B The DSM identifies the following disorders as possible causes of organically-based mood symptoms: substances such as hallucinogens and PCP; endocrine disorders, such as hypo- or hyperthyroidism; carcinoma of the pancreas; viral illness; and structural disease of the brain, such as that caused by a stroke.

A 38 year-old woman is dependent on the prescription pain medications oxycodone and hydrocodone. She is referred to a physician's office for medication-assisted treatment of her opioid addiction. Which of the following is most likely to be administered in the induction phase of treatment? A.methadone B.buprenorphine C.naltrexone D.LAAM (l-alpha-acetyl-methadol)

Correct Answer is: B The Drug Treatment Act of 2000 allows doctors to treat opioid dependence in their practices with FDA-approved opioid medication. In 2002, the FDA approved two medications for use in opioid addiction treatment: buprenorphine monotherapy (Subutex) and a buprenorphine/naloxone combination (Suboxone). Buprenorphine, an opioid partial agonist, activates receptors to a lesser degree than full agonists (i.e., morphine and heroin) and its effects reach a ceiling effect at moderate doses - not increasing, even with increases in dosage. At low doses, it has enough agonist effect to enable opioid-addicted individuals to discontinue misuse of opioids without experiencing withdrawal symptoms. Under certain circumstances and in high doses, it can block the effects of full opioid agonists and precipitate opioid withdrawal syndrome like an opioid antagonist. Buprenorphine carries a lower risk of abuse, addiction, and side effects than full agonists. Subutex (buprenorphine) is more often used at the beginning of treatment and withdrawal syndrome can be precipitated in individuals maintained on it. Suboxone, containing the opioid antagonist naltrexone*, was designed to decrease the potential for abuse by injection and is more often used in the maintenance treatment of opiate addiction. Both methadone*and LAAM (l-alpha-acetyl-methadol)* are effectively used to treat opioid addiction (* incorrect options); however they are not available in practice settings other than Opioid Treatment Programs (OTPs) (i.e., methadone clinics).

The primary associated feature of the Somatoform Disorders is: A.panic attacks. B.anxiety and depression. C.addiction to analgesics or mild tranquilizers. D.somatic delusions.

Correct Answer is: B The Somatoform Disorders (such as Somatization Disorder, Conversion Disorder, Somatoform Pain Disorder, Hypochondriasis, and Body Dysmorphic Disorder) are characterized by physical symptoms which have no known physical cause and are believed to be caused by psychological factors. The DSM identifies anxiety and depression as associated features of each of these disorders. You might have gone for addiction to analgesics or tranquilizers, since it seems to make sense. However, excessive use of analgesics (which are not addictive anyway) is identified as a possible associated feature of Somatoform Pain Disorder only.

A mother complains that her 10-year-old son has been misbehaving a lot lately. He refuses to do anything he is asked, has been staying out late at night, starting fights with his classmates, and has been caught lying on numerous occasions. The mother states that his misbehavior began about 18 months ago when his parents separated. Based on these symptoms, his most likely diagnosis is: Select one: A.Adjustment Disorder with Disturbance of Conduct B.Conduct Disorder C.Oppositional Defiant Disorder D.Child Antisocial Behavior

Correct Answer is: B The boy's symptoms meet the criteria for Conduct Disorder. There is a pattern of behavior in which major age-appropriate norms or rules are violated (staying out late at night, starting physical fights with others, lying, etc.). His symptoms are more severe than is characteristic of Oppositional Defiant Disorder and are not just isolated incidents of antisocial behavior as seen in Child Antisocial Behavior.

One of the highest correlations with smoking cessation outcome is related to Select one: A.gender. B.level of dependence. C.age. D.duration of smoking.

Correct Answer is: B The greater the level of dependence on nicotine, the harder it is to stop smoking (American Psychiatric Association, Practice Guidelines for the Treatment of Nicotine Dependence, Washington, DC, 1996). In terms of gender*, and age*, females and males and older and younger adults generally do equally well in treatment. The duration of smoking* is not really a factor (* incorrect options); it is far more critical to determine the amount of smoking in order to determine dependence.

Approximately what percent of women experience Major Depressive Disorder after childbirth. Select one: A.1 to 5%. B.10 to 20%. C.20 to 30%. D.30 to 40%.

Correct Answer is: B The majority of women experience some depression following childbirth but, for most, these symptoms are mild. For about 10 to 20% of women, symptoms are sufficiently severe to qualify for a diagnosis of Major Depression.

The risk of a monozygotic twin of a Schizophrenia proband developing Schizophrenia is approximately: Select one: A.0.25 B.0.5 C.0.75 D.1

Correct Answer is: B The term "proband" refers to the index population, that is, those persons identified with a particular disorder. Studies have found that the risk for a monozygotic (identical) twin of a schizophrenic proband to be diagnosed with Schizophrenia is about 46%. The risk for dizygotic (fraternal) twins is about 17%.

With a diagnosis of Schizophrenia, the risk of the same diagnosis for a monozygotic twin is how many times greater than the risk for a dizygotic twin? Select one: A.one time B.two and a half times C.four times D.six and a half times

Correct Answer is: B This is one of those questions that seems to be a favorite of the EPPP. The risk for an identical (monozygotic) twin to be diagnosed is about 46% and for fraternal (dizygotic) twins it is about 17%. You have to be able to do the basic math here, and realize that 46 is about two and a half times 17, and thus is the "closest" to correct. This question is also a good example of how the exam takes a straightforward finding and muddies the water with it.

The research has shown that, in the treatment of nicotine dependence, adding a behavioral intervention to nicotine replacement therapy Select one: A.has little or no effect in terms of either short- or long-term abstinence. B.has little additional effect in terms of short-term abstinence but does improve long-term abstinence. C.has some additional effect in terms of short-term abstinence but doesn't improve long-term abstinence. D.has a substantial effect on both short- and long-term abstinence

Correct Answer is: B This question is difficult since the research on this issue has been inconsistent. However, there is evidence that a combined treatment (behavioral intervention plus nicotine replacement therapy) is best, especially in terms of long-term abstinence (abstinence for six months or more)

Which of the following statements is true about the results of meta-analyses investigating the effectiveness of cognitive therapy in the treatment of depression? Select one: A.Cognitive therapy is more effective than insight-oriented and interpersonal therapy, but less effective than anti-depressant medication. B.Cognitive therapy is more effective than insight-oriented therapy but equal in effectiveness to interpersonal therapy and anti-depressant medication. C.Cognitive therapy is more effective than anti-depressant medication and insight-oriented therapy, but equal in effectiveness to interpersonal therapy. D.Cognitive therapy is more effective than insight-oriented therapy, interpersonal therapy, and anti-depressant medication.

Correct Answer is: B Though some individual studies have found cognitive therapy to be the most effective treatment for depression, the results of meta-analyses (such as the one conducted by the National Institute of Mental Health) indicate that cognitive therapy, interpersonal therapy, and anti-depressant medications are about equal in efficacy in the treatment of depression. All three treatments are superior to psychodynamic, or insight-oriented, therapies.

An interpersonal therapist who is working with a client suffering from a prolonged grief reaction would attempt to Select one: A.connect the person's current grief reaction to losses of significant others in early childhood. B.help the client understanding how the loss is impacting the current therapeutic relationship. C.build up a positive relationship with the client and avoid discussing the loss until the termination phase of therapy. D.modify the client's maladaptive behaviors under the assumption that psychological change follows behavioral change.

Correct Answer is: B Though the correct answer is an imprecise description of what an interpersonal therapist would do, it is the best answer available. Interpersonal therapists believe that depression is caused and maintained by disturbances in early life, especially attachment disturbances. Rather than focusing on the past, however, they focus on the connection between the presenting problem and client's current relationships -- including the current relationship with the therapist. Grief reactions are a common focus of interpersonal therapy.

The behavior therapy technique for reducing arousal to inappropriate stimuli by first masturbating to orgasm while imagining appropriate stimuli and then continuing to masturbate while fantasizing about paraphilic images after orgasm is called: Select one: A.Orgasmic Reconditioning B.Satiation Therapy C.Systematic Desensitization D.Aversive Conditioning

Correct Answer is: B When treating individuals with paraphilias, behavior therapies are used based on the reasoning that maladaptive behavior has been learned and can therefore be unlearned. Orgasmic Reconditioning Orgasmic reconditioning instructs a client to begin masturbating while fantasizing about the inappropriate stimulus then switch from the paraphilic to more appropriate fantasies at the moment of masturbatory orgasm. Systematic Desensitization Systematic desensitization pairs slow, systematic exposure to anxiety-inducing situations with relaxation training. Aversive Conditioning Aversive conditioning substitutes a negative response for a positive response to inappropriate stimuli such as through pairing paraphilic urges with negative experiences, for example electric shocks or unpleasant odors.

The Mini Mental Status Exam (MMSE) is especially useful for: Select one: A.assessing reality testing in individuals with a Psychotic Disorder B.assessing level of consciousness in individuals with Delirium C.screening for Mental Retardation in children and adolescents D.screening for Dementia in older adults

Correct Answer is: D The Mini Mental Status Exam (MMSE) is a screening tool for cognitive functioning that assesses five areas of cognitive functioning: orientation, registration, attention and calculation, recall, and language. It is most useful for screening for Dementia in older adults.

A child diagnosed with Attention-Deficit/Hyperactivity Disorder is most likely to also be diagnosed with: Select one: A.Tourette's Disorder B.Conduct Disorder C.Enuresis D.Obsessive-Compulsive Disorder

Correct Answer is: BAttention-Deficit/Hyperactivity Disorder (ADHD) is highly comorbid with Conduct Disorder. Between 30% and 50% of children with ADHD also meet the criteria for Conduct Disorder, with the highest comorbidity rates among the two subtypes marked by hyperactivity-impulsivity (Hyperactive-Impulsive and Combined Types). The percentage of patients with Conduct Disorder who also have ADHD is nearly 70%. If you incorrectly selected Tourette's Disorder, you may have been thinking of the reverse relationship. That is, among those with Tourette's Disorder the comorbidity of ADHD is at least 50%; however, most patients with ADHD do not have Tourette's Disorder.

A number of books in the popular press have been written regarding the relationship between psychological factors and cancer. Which of the following statements best reflects the outcome of scientific studies of this issue? Select one: A.Psychological factors are related both to the onset of cancer and the success of recovery from it. B.Psychological factors are related to the onset of cancer but not to the success of recovery from it. C.Psychological factors are not related to the onset of cancer but are related to the success of recovery from it. D.Psychological factors are related to neither the onset of cancer nor the success of recovery from it.

Correct Answer is: C A number of theories regarding the relationship between psychological factors and the onset of cancer have been proposed. For instance, some authors have proposed that the "Type C" personality, typically described as a cooperative, unassertive patient who suppresses anger and complies with external authorities, is at a higher risk for cancer. However, most research shows that psychological factors and stressful events have a small or no effect on cancer incidence. By contrast, psychological factors do appear to be related to recovery from cancer. For instance, psychological treatments combining support and training in self-hypnosis are associated with higher survival rates and improved quality of life in cancer patients.

The presence of which of the following suggests the diagnosis is Acute Stress Disorder rather than Posttraumatic Stress Disorder? Select one: A.response to the traumatic stressor involved intense fear, helplessness, or horror B.a sense of reliving or reexperiencing the traumatic stressor C.dissociative symptoms occur during or immediately following a traumatic stressor D.increased symptoms of anxiety, poor concentration and irritability

Correct Answer is: C Acute Stress Disorder, is by definition, only appropriate when the duration of symptoms last at least two days and occur within one month of the extreme stressor or traumatic event. Posttraumatic Stress Disorder requires symptoms to persist for more than a month so this diagnosis cannot be made within the initial month following the traumatic stressor.

All of the following are associated with increased suicide risk in adolescents except: Select one: A.depression. B.interpersonal conflict. C.loss of a parent. D.exposure to suicidal behavior.

Correct Answer is: C All of the other choices are associated with increased risk. Pagliano (1995) found that "depression" was the most frequently reported risk factor in adolescent suicides. "Interpersonal conflict" with a parent or boyfriend/girlfriend often immediately precedes a completed suicide in adolescents (Brent, 1988). Gould (1990) found that "exposure to suicidal behavior" increased risk.

The second stage of Alzheimer's Dementia is characterized by: Select one: A.loss of implicit memory B.loss of short-term memory C.inability to perform complex tasks D.inability to recognize family or friends

Correct Answer is: C Alzheimer's Disease is often categorized into 3 stages. In Stage 1, which lasts 2-4 years, short-term memory loss begins. Patients in this stage frequently complain about forgetting where they placed things. In Stage 2, which lasts 2-10 years, there is further memory impairment (mostly explicit rather than implicit) and they begin having difficulty performing complex tasks, such as balancing a checkbook or going grocery shopping. They may get lost in familiar places and become apathetic. In the final stage -- Stage 3 -- which lasts 1-3 years, there is serious impairment in most areas. During Stage 3 they may lose the ability to speak and become unable to recognize family, friends, or even themselves. They lose all capacity to care for themselves and have difficulty walking, are incontinent, and are ultimately bedridden and often die of an opportunistic respiratory infection.

Research comparing anxiety over the lifespan indicates: A.younger adults are more likely to benefit from cognitive-behavioral therapy, whereas older adults are more likely to benefit from pharmacotherapy. B.younger adults are more likely to be underdiagnosed and older adults more likely to be misdiagnosed as having an anxiety disorder. C.younger adults are less likely to have comorbid symptoms of depression than older adults. D.younger and older adults are equally likely to perceive symptoms as result of physical health problems.

Correct Answer is: C Anxiety is the most common psychiatric disorder in older adults, with generalized anxiety disorder being the most prevalent anxiety disorder according to recent findings. Research indicates comorbid symptoms of depression frequently occur in all adults with anxiety, however occurs more often in older adults.

Parasomnias activate one or more physiological systems during the sleeping and waking cycle at an inappropriate time. Which of the following types of parasomnias involves an individual involuntarily grinding or clenching their teeth while sleeping? Select one: A.REM Sleep Behavior Disorder (RBD) B.somnambulism C.sleep bruxism D.sleep talking

Correct Answer is: C Parasomnias are undesirable motor, verbal, or experiential phenomena that occur as primary sleep events or secondary to systemic disease and are categorized as occurring in rapid eye movement (REM) sleep, non-rapid eye movement (NREM) sleep, or as types not related to a specific sleep state. Because it involves a partial arousal, an individual exhibits symptoms of being asleep and awake at the same time. Parasomnias are most commonly caused by biological factors, stress, depression and other related factors, they tend to be more common in children than adults, and in some cases, run in families. Sleep bruxism refers to when a person involuntarily grinds or clenches their teeth while sleeping, sometimes leading to wearing down the teeth and jaw discomfort. Treatment typically involves wearing a mouth guard during sleep. Other types of parasomnias include: somnambulism (sleepwalking), which tends to run in families; nightmares, nocturnal leg cramps, sleep talking, sleep enuresis (bedwetting), sleep paralysis, confusional arousals, sleep terrors (nigh terrors) and REM Sleep Behavior Disorder (RBD).

Michael has a high-pressure legal career that involves frequent altercations with opposing counsel and ongoing stress. He commonly reacts to the stress of anger by clenching his teeth and generally tensing up. His therapist suggests biofeedback and would most likely recommend which type of biofeedback? A.EDR B.EEG C.EMG D.HRV

Correct Answer is: C Biofeedback is the process of identifying physiological variables, or responses, for the purpose of helping an individual develop greater sensory awareness and is achieved by using electronic instrumentation to monitor responses then providing the information to the individual to improve their physiological control of responding. Electromyogram (EMG) biofeedback, measures impulses in the muscles and indicates the degree of relaxation or contraction/tension. It is commonly used for conditions such as stress, tension headaches, chronic pain, muscle stiffness, incontinence, urinary urgency and frequency, and when muscles are healing. Electrodermal response (EDR) biofeedback, also referred to as galvinic skin response training (GSR), measures skin surface changes, giving feedback on the relation between emotional state and the activity of the sympathetic system via sweat gland activity, and is utilized for stress and hyperhidrosis (excessive sweating). Electroencephalogram (EEG) or neurofeedback provides information on brainwave activity and patterns. It is often used in the treatment of attention deficit hyperactivity disorder, depression, and epilepsy to improve attention, reduce impulsivity and promote recovery from head injuries and strokes. Heart rate variability (HRV), sometimes referred to as electrocardiogram (ECG), biofeedback monitors heart rate and cardiac reactivity from sensors placed on a person's fingers or wrist. It is useful for managing stress, high blood pressure, anxiety, and heartbeat irregularities. Other recognized types of biofeedback include: thermal or skin temperature (ST) biofeedback, which involves skin temperature and blood flow control; Respiratory Feedback (RFB), which involves control of breathing type and frequency; and Respiratory Sinus Arrhythmia (RSA); which involves the synchronous control of heart rate and respiration, in which there is a small rise in heart rate during inhalation and a corresponding decrease during exhalation. Depending on the reason for biofeedback, more than one type is often used. For example in this question, Michael might also use EDR to control his general stress response in addition to EMG to control the muscle tensing and teeth clenching responses. Behavioral skills training such as relaxation training, guided imagery and stress-coping techniques are also frequently used in addition to the actual biofeedback.

According to current cognitive-behavioral theories, a central factor in the maintenance of anorexia nervosa symptom is Select one: A.intense fear of gaining weight. B.positive reinforcement from family and friends for weight loss. C.the need to control eating. D.a fear of sexuality and sexual maturity.

Correct Answer is: C Cognitive-behavioral therapy is commonly used to treat anorexia nervosa, and is empirically supported as an effective intervention. There are a few different cognitive-behavioral theories of the disorder, but they are similar. They emphasize two major factors in the maintenance of the disorder: an extreme need to control eating, and an extreme tendency to judge self-worth on the basis of body shape and weight. Treatment, after addressing the immediate need to begin eating and gaining weight, includes self-monitoring, examination and restructuring of distorted beliefs, and relapse prevention training.

Of the following, which is most likely to cause rapid and abrupt changes in cognitive functioning? Select one: A.Dementia of the Alzheimer's Type B.AIDS Dementia Complex C.Vascular Dementia D.Korsakoff's syndrome

Correct Answer is: C Dementia is characterized by multiple cognitive deficits such as memory impairment, language disturbances, and impairments in executive functioning that have a specific physiological cause--i.e., a medical condition or the persisting effects of a substance. Vascular Dementia is diagnosed in individuals who have Dementia judged to be caused by a cerebrovascular disease such as a stroke. Its onset is typically abrupt and it typically has a fluctuating, stepwise course characterized by rapid changes in cognitive functioning and orientation. By contrast, the course in Alzheimer's Dementia tends to be slow and progressive, with abilities deteriorating over the span of several years. In AIDS Dementia Complex, the course is also typically progressive, but punctuated by abrupt accelerations. Korsakoff's syndrome is dementia caused by a lack of thiamine in the brain, which is usually caused by chronic alcoholism and sometimes by severe malnutrition. Onset is variable--sometimes rapid and sometimes insidious.

15 year old Susan was initially diagnosed with Bulimia-Nervosa (Purging Type). Her self-evaluation is unduly influenced by her body shape and weight. She worries about gaining weight, has been binging and purging on a daily basis for almost a year and her weight has steadily dropped to less than 85% of a minimally normal level. Her therapist reassesses Susan's diagnosis. The salient feature to consider in the differential diagnosis of Anorexia-Nervosa (Binge-Eating/Purge Type) is her: A.fear of gaining weight or getting fat B.cognitive distortions associated with body image C.continuation of binging and purging despite weight loss D.denial of seriousness of current body weight

Correct Answer is: C Individuals with Bulimia-Nervosa, unlike those with Anorexia-Nervosa, Binge-Eating/Purge Type, are able to maintain body weight at or above a minimally normal weight for height and age. Anorexia-Nervosa is characterized by a refusal to maintain a minimal normal body weight, with the threshold of underweight being less than 85% of weight expected. Susan's weight falls below this threshold and her continuation of binging and purging despite weight loss meets the refusal criteria of Anorexia. Disturbances in perception of body shape and weight* (* incorrect options), and an over influence of weight and shape on self-evaluation are essential features of both disorders.

Which of the following is true about children with Learning Disorders? Select one: A.Co-existing disorders are no more common in these children then among members of the general population. B.Although children with Learning Disorders are more likely to have co-existing disorders, as adults, they do not differ from members of the general population in terms of psychopathology. C.Children with Learning Disorders have higher rates of some disorders and, as adults, continue to exhibit more problems than members of the general population. D.Children with Learning Disorders are less likely to have other mental disorders than members of the general population.

Correct Answer is: C It makes sense that children who have serious learning problems would continue to have difficulties as adults. Also, the studies have clearly found that children with Learning Disorders are at higher risk for certain other disorders in childhood including ADHD, Oppositional Defiant Disorder, Conduct Disorder, and Major Depression.

Some experts argue that panic attacks do not occur in young children because Select one: A.children do not experience the bodily sensations associated with panic. B.children do not think catastrophically. C.children are not capable of making internal interpretations of bodily sensations. D.children are not capable of predicting that certain external events may trigger bodily symptoms.

Correct Answer is: C Not all experts agree that children cannot experience panic attacks. However, those that do, for the most part, argue that children do not experience panic because they are cognitively incapable of making internal catastrophic interpretations (e.g., I'm going crazy) of bodily symptoms.

An elderly male displays paranoid ideations. Dementia and psychosis have been ruled out. The best intervention in this case would likely be to Select one: A.refer him to a psychiatrist for phenothiazines. B.challenge his dysfunctional thought disturbances. C.alter his environment to make it seem less threatening. D.explore his early experiences to determine the roots of the problem.

Correct Answer is: C Questions like this are certainly frustrating, since in real life you would conduct a comprehensive assessment of the person before planning interventions. For the exam, however, you must do your best with the information you have at hand.Paranoid ideations in elderly clients are often wholly or partially due to what is happening around them. For instance, elderly people often live in high crime areas, are victimized by unscrupulous businessmen, and are plotted against in various ways by family members. If the person is mildly memory impaired, depressed, and/or hearing impaired, these events can seem even more threatening and the ideations can intensify. Thus, of the choices listed, the best option would be to alter the environment to make it seem less threatening.

Researchers in the area of cognitive therapy have conducted studies regarding the predominant types and styles of cognition in different mental disorders. These studies have found all of the following to be distinguishing features of cognitions in depression and anxiety, except Select one: A.in depression, cognitions about hopelessness and failure are more common than in anxiety. B.depressed people are more likely to have absolute thoughts about negative themes; anxious people are more likely to have questioning thoughts about the future of events. C.anxious people tend to be self-absorbed; depressed people tend to be more absorbed with others. D.depressed people have enhanced recall of negative feedback; anxious people have enhanced recall of threatening situations.

Correct Answer is: C Self-absorption is characteristic of the cognitive and information processing style of both depressed and anxious individuals. The other choices about the differences between depressed and anxious individuals are true.

Orientation is most frequently measured by which of the following scales? A.Global Orientation and Amnesia Test B.Gross Orientation and Awareness Test C.Galveston Orientation and Amnesia Test D.Gollingberg Orientation and Awareness Test

Correct Answer is: C The Galveston Orientation and Amnesia Test (GOAT), which assesses temporal orientation primarily, was developed to serially evaluate cognition during the subacute stage of recovery from closed head injury. The scale measures orientation to person, place, and time, and memory for events preceding and following the injury.

The symptoms of Obsessive-Compulsive Disorder can be alleviated through cognitive-behavioral treatments and medication interventions that reduce activity in the Select one: A.reticular activating system. B.inferior colliculus. C.caudate nucleus. D.locus coeruleus.

Correct Answer is: C The caudate nucleus appears to be overactive in people diagnosed with OCD. L.R. Baxter reports that both behavioral interventions and drug therapy affect metabolic rate in the caudate nucleus The reticular activating system*, which you should remember by now, is involved in attention and arousal. The inferior colliculus*, controls auditory reflexes, and the locus coeruleus* may be associated with Depression and Panic Disorder

What is the approximate probability that a person with a brother diagnosed with schizophrenia (not an identical or fraternal twin) would also have schizophrenia? Select one: A.0.6 B.0.45 C.0.1 D.0.005

Correct Answer is: C The concordance rate for schizophrenia among siblings is about 10%. If you didn't know this, you might have been able to guess the correct answer. A rate of 60% or 45% probably should have seemed too high. And 0.5% is at the low end of estimates of the approximate lifetime prevalence rate of schizophrenia and therefore should have seemed too low to be the concordance rate among related individuals.

In pre-adolescence, Panic Disorder is Select one: A.impossible to distinguish from Separation Anxiety Disorder B.not diagnosed since pre-adolescents cannot exhibit any of the cognitive symptoms of Panic Disorder C.most likely manifested as chest pain, tachycardia, shortness of breath, and refusal to go to school D.most likely manifested as chest pain, tachycardia, shortness of breath, and a feeling of "going crazy"

Correct Answer is: C The onset for Panic Disorder typically occurs between late adolescence and the mid-30s. However, a small number of cases do begin in childhood. Although not all theorists agree on whether or not children have the cognitive ability to catastrophize, the research indicates that 6 to 12-year-olds are capable of experiencing panic attacks, typically manifested as chest pain, tachycardia, shortness of breath, and refusal to go to school.

Among individuals with Attention-Deficit/Hyperactivity Disorder, the deficits in attention are currently believed to be due to significant difficulty in: A.attending to all tasks B.attending to interesting tasks C.regulating attention D.regulating conflicting emotions

Correct Answer is: C The predominant theories regarding ADHD suggest that individuals with the disorder have difficulty regulating their attention. It is not that they pay too little attention; on the contrary, they often pay too much attention to too many things, many of which are not relevant.

The research has shown that the extent and severity of the long-term problems associated with ADHD are strongly related to whether or not the child also had: Select one: A.symptoms of depression. B.a learning disability. C.conduct problems. D.a "difficult" temperament.

Correct Answer is: C The studies have consistently shown that the combination of hyperactivity and conduct problems is most associated with antisocial behavior and other serious problems in adulthood.

According to the "iceberg profile" developed by Morgan, which of the following psychological characteristics displayed by successful athletes is typically above the general population mean? Select one: A.anxiety B.anger C.vigor D.confusion

Correct Answer is: C Through the use of psychological testing, Morgan (1978) found that athletes scored significantly lower in negative mental health traits such as tension, depression, anger, fatigue and confusion, but significantly higher for vigor compared to U.S. norms. He referred to the mood characteristics of accomplished athletes as the "iceberg profile." Morgan's presented correlation between exercise and a decrease in anxiety has been confirmed in subsequent studies in both serious and recreational athletes.

All of the following statements are true of Tourette's Disorder, except: Select one: A.onset of the disorder always occurs before the age of 18. B.both motor tics and vocal tics are present. C.uncontrollable outbursts of vulgar and obscene utterances are present in the majority of cases. D.the disorder is more common in males than females.

Correct Answer is: C Tourette's Disorder is a Tic Disorder that involves multiple motor tics (involuntary jerky movements) and vocal tics (vocal sounds such as grunts, barks, and clicks). uncontrollable outbursts of vulgar and obscene utterances are present in the majority of cases. Coprolalia, or involuntary utterances of obscenities, occurs in 10% or less of cases -- not the majority, which is why this option is correct (remember, the question is asking for the statement that isn't true). For a diagnosis to be made, onset of the disorder must be before age 18. The disorder is between 1.5 and 3 times more common in males than in females.

Treatment of Gender Dysphoria in children would be most likely to focus on A.acting as a role-model of gender appropriate behavior for the child. B.helping the person "come out" as a transgender individual. C.peer relationships and self-esteem. D.working with a medical professional who can prescribe medications to alter hormonal imbalances.

Correct Answer is: C Typical interventions for Gender Dysphoria focuses on social skills and self-esteem.

As a treatment for Obsessive-Compulsive Disorder, in vivo exposure: Select one: A.is effective only when maximal anxiety is invoked during exposure sessions. B.is most effective when exposure is massed (versus distributed) but involves a gradual increase in the intensity of anxiety arousal. C.is most effective when exposure is combined with response prevention. D.is most effective when exposure is distributed (versus massed) and is combined with response prevention.

Correct Answer is: C While in vivo exposure has been found superior to exposure in imagination for Simple Phobia and Agoraphobia, the difference is less clear for OCD (both seem effective). In addition, gradual exposure and maximal exposure (flooding) seem to be about equally effective as are massed or distributed exposure sessions. However, there is evidence that the benefits of exposure are maximized when it is combined with response prevention.

During the sixth month of group therapy, two co-leaders have a disagreement in session about how to run the group. According to Yalom, they should Select one: A.allow the senior therapist to take control of the group. B.divide the group into two sub-groups, each led by one of the co-leaders. C.resolve the conflict in front of the group. D.agree to discuss the conflict later, outside the presence of the group.

Correct Answer is: C Yalom believes that co-leaders should not express conflict in front of the group during the early stages of therapy. However, in his book The Theory and Practice of Group Psychotherapy, he notes that mature resolution of conflict in the later stages can serve as a healthy model for the group members.

All of the following are true regarding women and depression, except Select one: A.marriage reduces the risk of depression to a greater extent for men than it does for women. B.the more children a woman has, the more likely it is that she'll be depressed. C.women who have multiple roles (e.g., a job, children, a marriage) are more vulnerable to depression than women who don't. D.gender differences in coping style apparently are a reason why women are at a greater risk for depression than men.

Correct Answer is: C women who have multiple roles (e.g., a job, children, a marriage) are more vulnerable to depression than women who don't. This choice is not true; women who have multiple roles appear to be at lower risk for depression. This may be because they have many different support sources and outlets for their competence -- i.e., if their life is not going well in one area, they can compensate with success in other areas. The other choices, according to the APA Task Force Report on Women and Depression, are true. gender differences in coping style apparently are a reason why women are at a greater risk for depression than men. Regarding this choice, the report states that men may be at a lower risk than women because, in response to their problems, they are more likely to employ action and mastery (e.g., work, sports, going out with friends) strategies that distract them from their worries and give them a sense of power and control. Women, on the other hand, tend to brood and dwell on their problems.

Post-traumatic amnesia (PTA) refers to the impaired orientation or amnesia following a traumatic brain injury. The duration of PTA is considered to be: A.related to outcome only for children. B.unrelated to outcome. C.a poor predictor of outcome. D.a good predictor of outcome.

Correct Answer is: D A common symptom resulting from head injury is post traumatic amnesia (PTA), a pattern of mental disturbance characterized by memory failure for day-to-day events, disorientation, misidentification of family and friends, impaired attention and illusions. Although PTA can vary from hour to hour and day to day, its duration is most commonly used as a guide to the extensiveness of the damage. The duration of PTA appears to be a sensitive and reliable index of severity, with the longer the PTA the stronger the probability of extensive damage. While there is currently no universal agreement that PTA is a better or more sensitive predictor of outcome after traumatic brain injury than depth and duration of unconsciousness, many consider the duration of PTA the best indicator of traumatic brain injury severity and the most dependable marker of outcome prediction, even in mild cases (i.e., duration of less than one hour). PTA impaired orientation may include retrograde and/or anterograde amnesia and researchers have suggested that PTA would be more accurately called posttraumatic confusional state. With respect to recovery, the duration of retrograde amnesia usually progressively declines while anterograde memory is frequently the last function to return especially after the recovery from loss of consciousness. Most individuals with a mild brain injury recover cognitive and behavioral functions and resolution of other symptoms within 3 to 6 months, though some continue to have symptoms for an extended period and individuals with a moderate or severe injury are likely to experience long-term symptoms and impairments in multiple areas of functioning. An increased risk for long-term impairment include: female gender, previous head trauma, and history of a neurological or psychiatric problem.

Tourette's Disorder is most associated with which of the following neurotransmitters? A.GABA B.serotonin C.acetylcholine D.dopamine

Correct Answer is: D Although the research findings are mixed, most studies have focused on the role of dopamine in Tourette's Disorder. The findings suggest that either an excess of dopamine or an increased sensitivity of dopamine receptors is related to Tourette's. Other studies have also linked serotonin and norepinephrine to the disorder. Antipsychotic medications such as Haloperidol (Haldol) are often used to control the tics in Tourette's Disorder by affecting the dopamine level.

Anxiety and depression share which of the following symptoms? Select one: A.anhedonia B.somatic arousal C.obsessive thoughts D.negative affect

Correct Answer is: D Several researchers have suggested that a general distress factor, sometimes referred to as "negative affect," is a core symptom in both anxiety and depression. Anhedonia* is unique to depression and somatic arousal* is unique to anxiety disorders. Obsessive thoughts* is a characteristic of Obsessive-Compulsive Disorder, but it is not a universal symptom in all of the anxiety disorders or the depressive disorders (* incorrect options).

Research on the cognitive effects of chemotherapy and radiation on children with cancer indicate: Select one: A.cognitive abilities are affected only during the course of treatment with both radiation and chemotherapy. B.cognitive abilities are affected only during the course of treatment with radiation, whereas chemotherapy is associated with developing cognitive problems following treatment. C.young boys are at greater risk to have cognitive problems than girls. D.younger children are at greater risk for more problems and more severe problems than are older children.

Correct Answer is: D Between infancy and 15 years of age, cancer is the leading cause of death by disease among U.S. children. Among the 11 major types of childhood cancers, leukemias (blood cell cancers), brain and other central nervous system (CNS) tumors account for over half of new cases. Treatment for childhood cancers can include chemotherapy, radiation, surgery, and stem cell transplants. Radiation and chemotherapy affect cognitive ability in children due to damage to the tiny blood vessels that carry nutrition and oxygen to the brain, resulting in calcifications; interference with the growing and thickening of the myelin; and with the growth and development of connecting nerve structures over time. Factors that increase the risk of long-term cognitive effects include diagnosis of cancer at a very young age, cancer treatment that results in reduced energy levels, cancer treatment that affects hearing or vision, cancer treatment that results in physical disabilities, cancer therapy that includes treatment to the central nervous system, numerous or prolonged school absences, a history of learning problems before being diagnosed with cancer. Research also indicates young girls are more vulnerable to lingering cognitive problems than boys, and children with acute lymphoblastic leukemia (ALL) and non-Hodgkin lymphoma are at a higher risk of developing later cognitive problems than those who have other forms of blood cancers. In fact, as many as 40% of all pediatric ALL patients treated with chemotherapy alone will develop serious learning disabilities within two to three years following treatment and for children who receive cranial radiation, with or without chemotherapy, the percentage is 80% to 90%. The most common cognitive problems found as a result of radiation and chemotherapy are with handwriting, spelling, reading or reading comprehension, understanding math concepts, attention deficits (tend to drift off and are easily distracted), short term memory and information retrieval, planning, and organizational skills, social maturity and social skills.

The tension-reduction hypothesis proposes that alcohol abuse is due to the effects of: Select one: A.tolerance B.withdrawal C.habituation D.conditioning

Correct Answer is: D Conger's (1956) tension-reduction hypothesis proposed that alcohol consumption reduces stress, which reinforces alcohol use, resulting in a greater likelihood of alcohol consumption in times of stress. Thus, the basis of the theory is operant conditioning [J. Conger, Reinforcement theory and the dynamics of alcoholism, Quarterly Journal of Studies on Alcohol, 1956, 17, 296-305]. Subsequent research findings on Conger's theory have been mixed, leading some researchers to conclude that the theory is overly simplistic.

Autistic children tend to do as well as or better than other children the same age on tests of Select one: A.abstract problem solving. B.facial recognition. C.response speed. D.field independent processing.

Correct Answer is: D Field dependence-independence is a construct viewed as a dimension of cognitive style, or the way in which individuals think, perceive, remember, and use information to solve problems. Those with a field independent cognitive style tend to use internal referents to solve problems and tend to see environments in terms of their constituent parts rather than as organized wholes. By contrast, a field dependent style relies on external or environmental referents to solve problems; perception tends to be dominated by the overall organization of the environment (or field) rather than its individual parts. Autistic individuals tend to have a field-independent cognitive style, and on some tasks that assess field independence, they consistently outperform their same-age peers. For example, numerous studies have found that autistic children outperform age-matched children on the Childhood Embedded Figures Test, which requires examinees to identify a simple figure hidden in a complex background design. Good performance on this test indicates field independence because it requires separating an item from the field in which it is embedded.

Functional nocturnal enuresis Select one: A.is associated with some physical medical disorder. B.most often occurs during REM sleep and is associated with dreaming. C.is not found in children above 12 years of age. D.is not associated with any particular stage of sleep.

Correct Answer is: D If the wetting is associated with a physical cause, then it is not diagnosed as "functional" enuresis. This disorder is not associated with dreaming REM-sleep as one might expect. It actually is not significantly correlated with any particular sleep stage, although it is most likely to occur during the first third of the night. And, while it is less common in older than in younger children, it does exist in older children and adolescents.

All of the following are factors that typically distinguish Dissociative Amnesia from amnesia due to known physical causes, except Select one: A.in Dissociative Amnesia, memory loss is primarily for autobiographical information. B.in Dissociative Amnesia, cognitive abilities are usually preserved. C.in Dissociative Amnesia, memory loss can be reversed. D.in Dissociative Amnesia, memory impairment is typically limited to information occurring immediately before an emotional trauma.

Correct Answer is: D In Dissociative Amnesia, memory loss is usually for information acquired after the emotional trauma that brings on the symptoms. There is typically a gap or a series of gaps in recall for the individual's life history. By contrast, in some forms of Amnestic Disorder (e.g., in Amnestic Disorder Due to a Brain Injury), memory loss is typically for information in a circumscribed period of time immediately before the injury occurs. All of the other choices describe features that typically distinguish between Dissociative Amnesia and amnesia due to known physical causes.

Blood-Injection-Injury Type of Specific Phobia, relative to other types of Specific Phobias, is characterized by: Select one: A.dry skin B.childhood onset C.awareness the fear is irrational D.low blood pressure Feedback

Correct Answer is: D In most of the Specific Phobias there is an increase in heart rate and blood pressure. However, in the Blood-Injection-Injury Type there is an initial brief acceleration in heart rate followed by a deceleration and a drop in blood pressure. This often results in vasovagal fainting. Due to this unique physiological response to the feared stimulus, the recommended treatment involves tensing muscles, rather than relaxing them, in the presence of the feared stimulus. Although the Blood-Injection-Injury Type usually does begin in childhood*, so do many of the other Specific Phobias. In all Specific Phobias the person recognizes that the fear is excessive* or unreasonable, except in children who may not recognize this (* incorrect options).

Some studies indicate that the rate of ADHD symptoms of hyperactivity and impulsivity among identical twins with ADHD to be Select one: A.0.2 B.0.4 C.0.6 D.0.8

Correct Answer is: D In terms of the role of genetics, some studies indicate higher rates of ADHD among relatives of afflicted individuals. For example, Biederman (1995) found that when a parent had childhood-onset ADHD, the risk for this diagnosis among his or her offspring was 57%. Additionally, a review of twin studies (Stevens, 1994) indicated that the average heritability is .80 for hyperactivity and impulsivity.

The presence of which of the following suggests the diagnosis is Social Phobia rather than Agoraphobia? Select one: A.social situations are avoided due to fear of humiliation or embarrassment B.an absence of the physiological symptoms associated with a panic attack C.anxiety that is perceived as excessive and temporarily controllable D.increased anxiety in feared situations when accompanied by a trusted companion

Correct Answer is: D Individuals with Agoraphobia typically prefer to be with a trusted companion when in the feared situation, whereas, individuals with Social Phobia may feel more anxious due to the potential scrutiny by a companion. social situations are avoided due to fear of humiliation or embarrassment This choice does not adequately differentiate the disorders since in both Social Phobia and Agoraphobia there is a fear of humiliation or embarrassment and an avoidance of those situations, although in Agoraphobia the fear is not limited to social situations.

A headache that begins in the frontal or occipital area of the head and then proceeds to involve a bilateral dull ache is: Select one: A.a common migraine. B.a classic migraine. C.a cluster headache. D.a tension headache.

Correct Answer is: D Migraine headaches cause unilateral throbbing pain. Cluster headaches cause intense pain behind the eye (which are the incorrect responses). So you're left with "tension headache," which is the correct answer.

Which of the following smoking cessation treatments is least effective for long-term abstinence? Select one: A.hypnosis B.acupuncture C.aversive techniques D.nicotine replacement

Correct Answer is: D Nicotine replacement interventions (e.g., nicotine gum, nicotine patch) have not been found to be very effective over the long-term unless they are combined with other treatment interventions.

The psychiatric diagnosis most associated with completed suicide is Select one: A.Bipolar I Disorder, Most Recent Episode Depressed. B.Bipolar Disorder, Most Recent Episode Mixed. C.Major Depressive Disorder, Recurrent. D.Major Depressive Disorder, Recurrent, With Psychotic Features.

Correct Answer is: D Of all the diagnostic categories, Mood Disorders are associated with the highest risk of completed suicide -- approximately 15% to 20% of individuals with Mood Disorders eventually commit suicide. And individuals with depressed and psychotic features are at five times greater risk for suicide as compared to patients with other Mood Disorders.

In comparison studies of younger and older adults, it has been found that depression in older adults is least likely to result in Select one: A.difficulties with memory problems. B.anxiety feelings. C.feelings of hopelessness. D.expressed sadness.

Correct Answer is: D Older adults are less likely than younger adults to express feelings of depression or sadness. They are more willing to express feelings of hopelessness* and anxiety*. They are also more apt to have memory problems* (* incorrect options).

Which of the following parasomnias usually affects middle-aged or elderly individuals? Select one: A.night terrors B.sleep talking C.sleep walking D.REM sleep behavior disorde

Correct Answer is: D REM sleep behavior disorder is a parasomnia that occurs later in the night than NREM disorders, usually affects middle-aged or elderly individuals, especially males, and sufferers often also have a neurological disorder. In this disorder, the temporary muscle paralysis that normally occurs during REM sleep does not occur so individuals may act out dreams through potentially violent movements or behaviors during sleep that can cause injuries to themselves or bed partners. NREM disorders occur early in the night and include somnambulism, or sleepwalking*, sleep talking*, enuresis (bed-wetting), and night, or sleep, terrors* (* incorrect options). These are all more common in children.

Research utilizing functional brain imaging techniques has shown a biological basis for Attention Deficit/Hyperactivity Disorder (ADHD) and has linked it to abnormalities in the following brain structures: Select one: A.parietal lobe, hypothalamus, and corpus callosum B.parietal lobe, striatum, and amygdala C.frontal lobe, thalamus, and amygdala D.frontal lobe, striatum, and cerebellum

Correct Answer is: D Recent research has established a biological basis for Attention Deficit/Hyperactivity Disorder (ADHD) with abnormalities in the right frontal lobe, striatum, and cerebellum most consistently implicated in this disorder. Other areas of the brain, including certain regions of the parietal lobe, have been linked to ADHD to a lesser extent. Using the core symptoms of ADHD and functions of the major brain structures, you may have determined the frontal lobes (mediates higher-order functions), the striatum (part of the basal ganglia and composed of the caudate nucleus and the putamen) and cerebellum (involved in motor activity) are the areas linked to this disorder.

What percentage of children diagnosed with Attention-Deficit/Hyperactivity Disorder continue to exhibit signs of the disorder in adulthood? Select one: A.5 B.10 C.25 D.50

Correct Answer is: D Research studies have found that between 30% and 70% of children with ADHD continue to exhibit signs of the disorder throughout their lives.

The differential diagnosis between Schizoid Personality Disorder and Avoidant Personality Disorder is based on Select one: A.degree of isolation. B.odd behaviors. C.self-centeredness. D.fear of rejection.

Correct Answer is: D Schizoid Personality Disorder is characterized by a pattern of indifference to social relations and a limited range of emotional expression in social situations. Avoidant Personality Disorder is characterized by social inhibition, feelings of inadequacy, and hypersensitivity to negative evaluation. Individuals with both these disorder are likely to avoid social relationships. However, those with Avoidant Personality Disorder do so due to timidity and fear of criticism, disapproval, or rejection. Individuals with Schizoid Personality Disorder, by contrast, do so out of indifference to social relationships and a preference for solitary activities.

The most common diagnosis for inpatients in the 18-44 age range is Select one: A.alcoholism. B.mania. C.depression. D.Schizophrenia.

Correct Answer is: D Schizophrenia is the most common diagnosis for inpatients in the age 18 to 44 range.

Which of the following is least likely to cause secondary impotence? Select one: A.medication use B.alcohol use C.diabetes mellitus D.old age

Correct Answer is: D Secondary impotence is diagnosed when a man persistently or recurrently fails to attain or maintain an erection even though in the past he has successfully achieved an erection. The erectile reflex is usually unimpaired in older men; therefore, aging alone is not likely to be a cause of impotence. The other responses are common physical causes of secondary impotence

A high level of expressed emotion by family members has been found to be predictive of relapse for which of the following disorders? Select one: A.schizophrenia B.eating disorders C.mood disorders D.all of the above

Correct Answer is: D The relationship between a family's level of expressed emotion (EE) and the risk of relapse among patients with Schizophrenia has been known for many years; however, recently EE has also been linked to eating disorders and mood disorders. In fact, in a meta-analysis the relationship of EE to relapse in eating disorders and mood disorders was significantly greater than the relationship found for Schizophrenia

Of the following, the strongest sign that an adolescent is about to attempt suicide would be Select one: A.veiled non-specific statements that may be interpreted as a threat, such as "I can't take anymore" B.negative symptoms of depression such as restricted range of affect and psychomotor retardation. C.wild mood swings characterized by periods of euphoria and non-depression. D.social isolation and withdrawal in response to a recent psychosocial stressor.

Correct Answer is: D This question is difficult because anytime a person is depressed, the possibility of suicide must be assessed and ruled out, and all of the choices include symptoms of depression. However, there are some specific warning signs of elevated suicide risk that are more common and more dangerous than others, and of the choices available, this one ("social isolation and withdrawal in response to a recent psychosocial stressor") best fits in this category. One such sign is a sudden change in behavior in response to a recent loss or stressor; another is depression characterized by isolation and withdrawal from family and friends. Additional signs of elevated risk include 1) specific threats (as opposed to general threats like the one in the first choice, though those cannot be dismissed either); 2) giving away valued possessions; 3) getting one's affairs in order; 4) self-destructive and risk-taking behaviors and 5) a history of past suicidal behavior. Certainly, the symptoms described by the other choices may be present as well, but they are not associated with an elevated risk of suicide to the degree that social isolation in response to stressor is.

Symptoms of premenstrual dysphoric disorder are prominent: 1. immediately preceding and during the first few days of menses. 2. during menses. 3. during ovulation. 4. during and immediately following menses.

Correct Answer: 1. immediately preceding and during the first few days of menses Feedback: According to the DSM-5, symptoms of premenstrual dysphoric disorder must include one of the following: marked affective lability, marked irritability or increased interpersonal conflict, marked depressed mood, or marked anxiety. Additionally, at least one of the following must be present (to reach a total of five symptoms): decreased interest in usual activities, difficulty concentrating, lethargy or fatigue, change in appetite, insomnia or hypersomnia, feeling overwhelmed, and physical symptoms (e.g., breast tenderness, joint/muscle pain, bloating, or weight gain). Symptoms are present in the week before the onset of menses, symptoms start to improve within a few days after the onset of menses, and symptoms are minimal or absent in the week postmenses. If you lacked information about this disorder, given that the name includes "premenstrual" you might have been able to deduce that symptoms are present prior to menstruation.

Schizoaffective disorder involves: 1. alternating periods of major mood episodes with symptoms of schizophrenia. 2. the concurrent presence of a major mood episode with symptoms of schizophrenia; additionally there are delusions or hallucinations present for at least two weeks without mood symptoms. 3. the concurrent presence of symptoms of schizophrenia with a major mood episode; additionally there is a major mood episode for at least two weeks without delusions or hallucinations. 4. a form of schizophrenia in which mood symptoms are very prominent.

Correct Answer: 2. the concurrent presence of a major mood episode with symptoms of schizophrenia; additionally there are delusions or hallucinations present for at least two weeks without mood symptoms. Feedback: DSM-5 diagnostic criteria for schizoaffective disorder include an uninterrupted period of illness during which there has been a major mood episode (e.g., major depressive or manic) concurrent with the symptoms of schizophrenia. In addition, during that period of illness, there have been delusions or hallucinations for at least 2 weeks without prominent mood symptoms. A key aspect of the DSM-5 definition is that mood and psychotic symptoms need to be concurrent (ruling out Response 1). Response 3 is the exact reverse of the criteria. Schizoaffective disorder is not a form of schizophrenia (Response 4); rather it is considered a schizophrenia spectrum and other psychotic disorder. HINT: the disorder's name "schizoaffective" suggests that the "schizo" or symptoms of schizophrenia predominate.

Which of the following is the most accurate statement regarding selective mutism? 1. It is a communication disorder in the DSM-5 category of neurodevelopmental disorders. 2. It is typically an act of defiance and may be effectively treated with behavior modification. 3. It is an anxiety disorder and may be treated with guided imagery. 4. It results from trauma and treatment typically involves therapy focused on overcoming the trauma.

Correct Answer: 3. Feedback: Selective mutism (SM) involves a consistent failure to speak in specific social situations when speaking is expected (e.g., the classroom), in spite of speaking in other situations (e.g., at home). As a result, this disturbance interferes with educational or occupational achievement, or with social communication. Selective mutism is an anxiety disorder. The failure to speak in children with selective mutism (SM) is related to fear and anxiety, not willfulness or disobedience (ruling out Response 2). Typical treatments include some combination of behavioral therapy, cognitive behavioral therapy, individual therapy, family therapy, and medication. Behavioral therapy for SM usually includes desensitization and relaxation protocols, of which guided imagery is sometimes a part (Response 3, correct answer). SSRIs (e.g., Paxil) are commonly prescribed for their anti-anxiety effects, although other antidepressants are sometimes used. Selective mutism is not considered a communication disorder, an example of which includes childhood-onset fluency disorder (stuttering). The communication disorders are listed in the DSM-5 category of neurodevelopmental disorders (Response 1). Selective mutism is also not related to a history of trauma (ruling out Response 4).

Which of the following is included among the diagnostic criteria for separation anxiety disorder? 1. Social withdrawal and sadness. 2. Panic attacks outside of the home. 3. Worry about the safety of the parents or primary caretakers. 4. Onset before age 18.

Correct Answer: 3. Worry about the safety of the parents or primary caretakers. Feedback: According to DSM-5, the essential feature of separation anxiety disorder is developmentally inappropriate and excessive anxiety concerning being away from home or away from the person(s) to whom the individual is attached. Among the diagnostic criteria is worry about harm befalling one's major attachment figure. Social withdrawal and sadness (Response 1), as well as panic attacks outside the home (Response 2), may be associated features of the disorder but are not part of the diagnostic criteria. The DSM-5 allows for the diagnosis of separation anxiety disorder in adults (Response 4).

Mr. Veeray Nervous requests behavior therapy. He reports that he's terrified of public speaking. In response to your questions, he admits that he has had numerous panic attacks when he has attempted to speak publicly. Your most likely diagnosis would be: • 1. Agoraphobia. • 2. Specific Phobia, Situational Type. • 3. Social Anxiety Disorder • 4. Panic Disorder.

Correct Answer: 3. social anxiety disorder. Feedback: Social anxiety disorder is diagnosed when there is fear of one or more social or performance situations. In this case, the fear is of public speaking. An individual with social anxiety disorder may experience panic attacks when exposed to the feared situation. In contrast, panic disorder (Response 4) is characterized by recurrent unexpected panic attacks. In this example, the panic attacks occur only when the patient is public speaking. Specific phobia (Response 2) cannot be diagnosed in situations that are feared solely because of negative evaluation. Agoraphobia (Response 1) involves intense fear or anxiety about two or more situations: using public transportation, being in open spaces, being in enclosed spaces, standing in line or being in a crowd, or being outside the home. These situations are avoided because of thoughts that escape might be difficult or that help might not be available when incapacitating or embarrassing symptoms occur.

A 22-month-old child is brought in by her mother. Her mother notes that while her daughter is verbally precocious, speaking in full sentences, she is concerned because her daughter stutters. The most likely diagnosis is: 1. language disorder. 2. childhood-onset fluency disorder. 3. speech sound disorder. 4. no disorder.

Correct Answer: 4. no disorder. Feedback: According to the DSM-5, language disorder (Response 1) involves difficulty acquiring and using language due to deficits in comprehension or production. Individuals with language disorder present with limited vocabulary, limited sentence structure, and impairments in discourse. Speech sound disorder (Response 3) involves difficulty with phonology and articulation that interferes with intelligibility or prevents verbal communication. Examples include errors in sound production and use, including substituting one sound for another or omitting sounds such as final consonants. Childhood-onset fluency disorder (stuttering) is diagnosed when there is a disturbance in the normal fluency and time patterning of speech, characterized by sound and syllable repetitions, interjections, broken words, blocking, circumlocutions, and monosyllabic whole-word repetitions. Stuttering (Response 2) is considered to be normal until about age 2 and would not warrant a diagnosis until after that age. The most accurate response, therefore, would be to give the child no diagnosis (Response 4).

In treating a client, a therapist using Lynn Rehm's Self-Control Model of Depression would be most likely to employ: A.exercises to assist the client in setting less stringent criteria for evaluating the client's behavior. B.daily charting of depressogenic attributions. C.practicing control of emotional reactions. D.experiments to identify automatic thoughts.

Correct Answer: A - exercises to assist the client in setting less stringent criteria for evaluating the client's behavior. Rehm's (Rehm, Kaslow, & Rabin, 1987) self-control model attributes depression to problems related to self-monitoring, self-evaluation, and self-reinforcement. According to this model, people who are depressed attend more to negative than positive life events, have strict standards of self-evaluation and make internal attributions for negative events, and provide themselves with insufficient reinforcement and excessive punishment.


Conjuntos de estudio relacionados

National Topic Tester - Laws of Agency

View Set

Chapter 2 - Concept of Teaching and Learning

View Set